Last visit was: 23 Apr 2024, 12:07 It is currently 23 Apr 2024, 12:07

Close
GMAT Club Daily Prep
Thank you for using the timer - this advanced tool can estimate your performance and suggest more practice questions. We have subscribed you to Daily Prep Questions via email.

Customized
for You

we will pick new questions that match your level based on your Timer History

Track
Your Progress

every week, we’ll send you an estimated GMAT score based on your performance

Practice
Pays

we will pick new questions that match your level based on your Timer History
Not interested in getting valuable practice questions and articles delivered to your email? No problem, unsubscribe here.
Close
Request Expert Reply
Confirm Cancel
SORT BY:
Date
Tags:
Show Tags
Hide Tags
Math Expert
Joined: 02 Sep 2009
Posts: 92881
Own Kudos [?]: 618572 [14]
Given Kudos: 81562
Send PM
Director
Director
Joined: 25 Jul 2018
Posts: 668
Own Kudos [?]: 1117 [0]
Given Kudos: 69
Send PM
Senior Manager
Senior Manager
Joined: 14 Jul 2019
Status:Student
Posts: 478
Own Kudos [?]: 369 [0]
Given Kudos: 52
Location: United States
Concentration: Accounting, Finance
GMAT 1: 650 Q45 V35
GPA: 3.9
WE:Education (Accounting)
Send PM
Senior Manager
Senior Manager
Joined: 28 Feb 2014
Posts: 471
Own Kudos [?]: 558 [1]
Given Kudos: 74
Location: India
Concentration: General Management, International Business
GMAT 1: 570 Q49 V20
GPA: 3.97
WE:Engineering (Education)
Send PM
Re: Is x - x^2 > y - y^2 ? (1) x > y (2) x^2 > y^2 [#permalink]
1
Kudos
Bunuel wrote:
Is \(x - x^2 > y - y^2\) ?


(1) \(x > y\)

(2) \(x^2 > y^2\)


x - x^2 > y - y^2
x - y > x^2 - y^2
x - y > (x - y) (x + y)
(x - y) ( x + y -1) < 0

(1) x > y
x - y >0; ( x + y -1) can be positive or negative. Insufficient

(2) x^2 > y^2; taking square root on both the sides
|x| > |y|; signs of x and y are unknown. Insufficient

(1)+(2) still signs of x and y are unknown. Insufficient

E is correct.
Math Revolution GMAT Instructor
Joined: 16 Aug 2015
Posts: 10161
Own Kudos [?]: 16592 [2]
Given Kudos: 4
GMAT 1: 760 Q51 V42
GPA: 3.82
Send PM
Re: Is x - x^2 > y - y^2 ? (1) x > y (2) x^2 > y^2 [#permalink]
2
Bookmarks
Expert Reply
Bunuel wrote:
Is \(x - x^2 > y - y^2\) ?


(1) \(x > y\)

(2) \(x^2 > y^2\)


Are You Up For the Challenge: 700 Level Questions



Forget conventional ways of solving math questions. For DS problems, the VA (Variable Approach) method is the quickest and easiest way to find the answer without actually solving the problem. Remember that equal numbers of variables and independent equations ensure a solution.
Visit https://www.mathrevolution.com/gmat/lesson for details.

The first step of the VA (Variable Approach) method is to modify the original condition and the question. We then recheck the question. We should simplify conditions if necessary.

The question is equivalent to (x-y)(x+y-1) < 0 for the followings.

\(x - x^2 > y - y^2\)
\(⇔ 0 > x^2 - y^2 - x + y\)
\(⇔ x^2 - y^2 - x + y < 0\)
\(⇔ (x+y)(x-y) - (x-y) < 0\)
\(⇔ (x-y)(x+y-1) < 0\)

Since we have 2 variables (x and y) and 0 equations, C is most likely the answer. So, we should consider conditions 1) & 2) together first. After comparing the number of variables and the number of equations, we can save time by considering conditions 1) & 2) together first.

Conditions 1) & 2)

If \(x = 2, y = 1\), then \((x-y)(x+y+1) = 1 \cdot 2 = 2 > 0\) and the answer is 'no'.
If \(x = 2, y = -1.5\), then \((x-y)(x+y-1) = 3.5 \cdot (-0.5) < 0\) and the answer is 'yes'.

Since both conditions together do not yield a unique solution, they are not sufficient.

Therefore, E is the answer.

Normally, in problems which require 2 equations, such as those in which the original conditions include 2 variables, or 3 variables and 1 equation, or 4 variables and 2 equations, each of conditions 1) and 2) provide an additional equation. In these problems, the two key possibilities are that C is the answer (with probability 70%), and E is the answer (with probability 25%). Thus, there is only a 5% chance that A, B, or D is the answer. This occurs in common mistake types 3 and 4. Since C (both conditions together are sufficient) is the most likely answer, we save time by first checking whether conditions 1) and 2) are sufficient, when taken together. Obviously, there may be cases in which the answer is A, B, D, or E, but if conditions 1) and 2) are NOT sufficient when taken together, the answer must be E.
Manager
Manager
Joined: 04 Dec 2015
Posts: 186
Own Kudos [?]: 64 [0]
Given Kudos: 407
Send PM
Is x - x^2 > y - y^2 ? (1) x > y (2) x^2 > y^2 [#permalink]
Bunuel please help ! Is there any approach other than putting variables?

Posted from my mobile device

egmat chiranjeev please help!
GMAT Club Legend
GMAT Club Legend
Joined: 08 Jul 2010
Status:GMAT/GRE Tutor l Admission Consultant l On-Demand Course creator
Posts: 5957
Own Kudos [?]: 13375 [0]
Given Kudos: 124
Location: India
GMAT: QUANT+DI EXPERT
Schools: IIM (A) ISB '24
GMAT 1: 750 Q51 V41
WE:Education (Education)
Send PM
Is x - x^2 > y - y^2 ? (1) x > y (2) x^2 > y^2 [#permalink]
Expert Reply
Bunuel wrote:
Is \(x - x^2 > y - y^2\) ?


(1) \(x > y\)

(2) \(x^2 > y^2\)


Are You Up For the Challenge: 700 Level Questions


INSEADIESE

Question: Is \(x - x^2 > y - y^2\)?
Question: Is \(x - y > x^2 - y^2\)?

Even after combining the two statements

Case 1: x = 1/2 and y = 1/3 i.e. \(x - y > x^2 - y^2\)

Case 2: x = 3 and y = 2 i.e. \(x - y < x^2 - y^2\)

NOT SUFFICIENT

Answer: Option E
Manager
Manager
Joined: 10 Dec 2020
Posts: 76
Own Kudos [?]: 14 [0]
Given Kudos: 279
Concentration: Technology, Statistics
WE:Analyst (Computer Software)
Send PM
Re: Is x - x^2 > y - y^2 ? (1) x > y (2) x^2 > y^2 [#permalink]
MathRevolution wrote:
Bunuel wrote:
Is \(x - x^2 > y - y^2\) ?


(1) \(x > y\)

(2) \(x^2 > y^2\)


Are You Up For the Challenge: 700 Level Questions



Forget conventional ways of solving math questions. For DS problems, the VA (Variable Approach) method is the quickest and easiest way to find the answer without actually solving the problem. Remember that equal numbers of variables and independent equations ensure a solution.
Visit https://www.mathrevolution.com/gmat/lesson for details.



The first step of the VA (Variable Approach) method is to modify the original condition and the question. We then recheck the question. We should simplify conditions if necessary.

The question is equivalent to (x-y)(x+y-1) < 0 for the followings.

\(x - x^2 > y - y^2\)
\(⇔ 0 > x^2 - y^2 - x + y\)
\(⇔ x^2 - y^2 - x + y < 0\)
\(⇔ (x+y)(x-y) - (x-y) < 0\)
\(⇔ (x-y)(x+y-1) < 0\)

Since we have 2 variables (x and y) and 0 equations, C is most likely the answer. So, we should consider conditions 1) & 2) together first. After comparing the number of variables and the number of equations, we can save time by considering conditions 1) & 2) together first.

Conditions 1) & 2)

If \(x = 2, y = 1\), then \((x-y)(x+y+1) = 1 \cdot 2 = 2 > 0\) and the answer is 'no'.
If \(x = 2, y = -1.5\), then \((x-y)(x+y-1) = 3.5 \cdot (-0.5) < 0\) and the answer is 'yes'.

Since both conditions together do not yield a unique solution, they are not sufficient.

Therefore, E is the answer.

Normally, in problems which require 2 equations, such as those in which the original conditions include 2 variables, or 3 variables and 1 equation, or 4 variables and 2 equations, each of conditions 1) and 2) provide an additional equation. In these problems, the two key possibilities are that C is the answer (with probability 70%), and E is the answer (with probability 25%). Thus, there is only a 5% chance that A, B, or D is the answer. This occurs in common mistake types 3 and 4. Since C (both conditions together are sufficient) is the most likely answer, we save time by first checking whether conditions 1) and 2) are sufficient, when taken together. Obviously, there may be cases in which the answer is A, B, D, or E, but if conditions 1) and 2) are NOT sufficient when taken together, the answer must be E.


can't we modify the equation to x+y<1 ?

x-y>x^2-y^2
x-y > (x-y)(x+y)
1>x+y
Director
Director
Joined: 05 Jul 2020
Posts: 590
Own Kudos [?]: 301 [0]
Given Kudos: 154
GMAT 1: 720 Q49 V38
WE:Accounting (Accounting)
Send PM
Re: Is x - x^2 > y - y^2 ? (1) x > y (2) x^2 > y^2 [#permalink]
Vishalcv wrote:
MathRevolution wrote:
Bunuel wrote:
Is \(x - x^2 > y - y^2\) ?


(1) \(x > y\)

(2) \(x^2 > y^2\)


Are You Up For the Challenge: 700 Level Questions



Forget conventional ways of solving math questions. For DS problems, the VA (Variable Approach) method is the quickest and easiest way to find the answer without actually solving the problem. Remember that equal numbers of variables and independent equations ensure a solution.
Visit https://www.mathrevolution.com/gmat/lesson for details.



The first step of the VA (Variable Approach) method is to modify the original condition and the question. We then recheck the question. We should simplify conditions if necessary.

The question is equivalent to (x-y)(x+y-1) < 0 for the followings.

\(x - x^2 > y - y^2\)
\(⇔ 0 > x^2 - y^2 - x + y\)
\(⇔ x^2 - y^2 - x + y < 0\)
\(⇔ (x+y)(x-y) - (x-y) < 0\)
\(⇔ (x-y)(x+y-1) < 0\)

Since we have 2 variables (x and y) and 0 equations, C is most likely the answer. So, we should consider conditions 1) & 2) together first. After comparing the number of variables and the number of equations, we can save time by considering conditions 1) & 2) together first.

Conditions 1) & 2)

If \(x = 2, y = 1\), then \((x-y)(x+y+1) = 1 \cdot 2 = 2 > 0\) and the answer is 'no'.
If \(x = 2, y = -1.5\), then \((x-y)(x+y-1) = 3.5 \cdot (-0.5) < 0\) and the answer is 'yes'.

Since both conditions together do not yield a unique solution, they are not sufficient.

Therefore, E is the answer.

Normally, in problems which require 2 equations, such as those in which the original conditions include 2 variables, or 3 variables and 1 equation, or 4 variables and 2 equations, each of conditions 1) and 2) provide an additional equation. In these problems, the two key possibilities are that C is the answer (with probability 70%), and E is the answer (with probability 25%). Thus, there is only a 5% chance that A, B, or D is the answer. This occurs in common mistake types 3 and 4. Since C (both conditions together are sufficient) is the most likely answer, we save time by first checking whether conditions 1) and 2) are sufficient, when taken together. Obviously, there may be cases in which the answer is A, B, D, or E, but if conditions 1) and 2) are NOT sufficient when taken together, the answer must be E.


can't we modify the equation to x+y<1 ?

x-y>x^2-y^2
x-y > (x-y)(x+y)
1>x+y


Hey Vishalcv, you can't simply cancel out (x-y) here as by doing so you will ignoring a lot of possible solutions. eg - if x=4 and Y=10, the equation holds true, but does that mean x+y <1? No.
Manager
Manager
Joined: 28 Nov 2020
Posts: 119
Own Kudos [?]: 25 [0]
Given Kudos: 96
Send PM
Re: Is x - x^2 > y - y^2 ? (1) x > y (2) x^2 > y^2 [#permalink]
fractions were my first choice 1/2 and 1/4 it's shows greater however when you add 1 and 0 it becomes equal even after eliminating negative , identifying zero is the key hence clearly insuff both statements IMO E
User avatar
Non-Human User
Joined: 09 Sep 2013
Posts: 32628
Own Kudos [?]: 821 [0]
Given Kudos: 0
Send PM
Re: Is x - x^2 > y - y^2 ? (1) x > y (2) x^2 > y^2 [#permalink]
Hello from the GMAT Club BumpBot!

Thanks to another GMAT Club member, I have just discovered this valuable topic, yet it had no discussion for over a year. I am now bumping it up - doing my job. I think you may find it valuable (esp those replies with Kudos).

Want to see all other topics I dig out? Follow me (click follow button on profile). You will receive a summary of all topics I bump in your profile area as well as via email.
GMAT Club Bot
Re: Is x - x^2 > y - y^2 ? (1) x > y (2) x^2 > y^2 [#permalink]
Moderator:
Math Expert
92881 posts

Powered by phpBB © phpBB Group | Emoji artwork provided by EmojiOne